¿Cómo se relaciona S=klnWS=kln⁡WS = k\ln W con la afirmación de que el calor no fluye de los objetos fríos a los calientes?

Hay varias formas en que se puede enunciar la Segunda Ley de la Termodinámica. Estoy pensando en estos dos:

La entropía nunca decrece espontáneamente.

y

El calor no fluye espontáneamente de los objetos fríos a los calientes.

De la primera afirmación, sabemos que la entropía del objeto está dada por S = k en W , dónde W es el número de microestados. ¿Cómo es esto equivalente a la segunda afirmación de que el calor no fluye espontáneamente de los objetos fríos a los calientes? La segunda declaración no parece estar relacionada con los microestados en absoluto.

Intenté dibujar dos sistemas, uno que tiene cinco niveles de energía cuántica accesibles (correspondientes a una temperatura más alta) y otro con solo dos niveles. El primer sistema tiene entonces cinco microestados diferentes y el segundo tiene solo dos. Si el calor fluyera de los objetos fríos a los calientes, entonces el segundo objeto tendría solo un nivel de energía mientras que el primero tendría seis. La entropía total es ahora k en 6 + k en 1 < k en 5 + k en 2 , es decir, la entropía ha disminuido, lo que hace que todo el enfoque parezca una tontería.

La conexión es proporcionada por Boltzmann H Teorema, que muestra que la definición estadística de entropía es (bajo condiciones genéricas) siempre no decreciente.
Creo que una respuesta completa a su pregunta podría ser simplemente un curso de mecánica estadística. Pero tenga en cuenta que su ejemplo no funciona, porque la temperatura depende del estado, no solo del sistema. Tienes que especificar números de ocupación para tus niveles de energía para hablar de una temperatura.
La ecuación de Boltzmann es para un sistema microcanónico (cerrado), mientras que la entropía no decreciente es para conjuntos de tipo canónico (pequeña porción de un gran sistema cerrado). Tienes que abreviar esos dos para responder la pregunta. No es necesario QM aquí, ya que ni siquiera usa sus propiedades distintivas, en comparación con la mecánica clásica.

Respuestas (1)

Una respuesta cualitativa a su pregunta es que un sistema a mayor temperatura tiene mayor W, siendo todo lo demás igual. Así que si tomas una cierta cantidad de calor q fuera de un sistema a alta temperatura T h , sin hacer nada más, el cambio de entropía es q T h = k Δ W h W h (dónde Δ W h < 0 ), y si pones ese mismo calor en un sistema a baja temperatura T yo , su cambio de entropía es + q T yo = k Δ W yo W yo ( Δ W yo > 0 ). Si suma estos, el cambio neto de entropía del sistema es proporcional a Δ W h W yo + Δ W yo W h . Porque el W son enormemente grandes y para pequeños incrementos de calor el Δ W son mucho más pequeños, esto es positivo si W h > W yo , cual es.

Su ejemplo no es la forma correcta de pensar al respecto. En cambio, desea considerar su sistema de 3 estados o de 5 estados a dos temperaturas diferentes, es decir, poblado por muchas partículas, pero con una energía promedio diferente para cada partícula. Por ejemplo, utilizando su sistema de 3 estados, W sería la cantidad de formas en que podría organizar N partículas en sus 3 niveles con una energía promedio determinada. Descubrirá que si toma algo de energía de los niveles de alta energía y la distribuye entre los niveles de baja energía, aumenta el número de formas de organizar las partículas, mientras que lo contrario es cierto para lo contrario.